Difference between revisions of "1992 AIME Problems/Problem 4"

m (Watch your vandalism, Herbs Li.)
(28 intermediate revisions by 14 users not shown)
Line 1: Line 1:
 
== Problem ==
 
== Problem ==
  
In Pascal's Triangle, each entry is the sum of the two entries above it. In which row of Pascal's Triangle do three consecutive entries occur that are in the ratio <math>3: 4: 5</math>?
+
In Pascal's Triangle, each entry is the sum of the two entries above it. In which row of [[Pascal's Triangle]] do three consecutive entries occur that are in the ratio <math>3: 4: 5</math>?
  
== Solution ==
+
== Solution 1==
  
In Pascal's Triangle, we know that the binomial coefficients of the nth row are <math>\displaystyle \binom{n}{0}</math>, <math>\displaystyle \binom{n}{1}</math>,...,<math>\displaystyle \binom{n}{n}</math>. Let our row be the nth row such that the three consecutive entries are <math>\displaystyle \binom{n}{r}</math>, <math>\displaystyle \binom{n}{r+1}</math>, and <math>\displaystyle \binom{n}{r+2}</math>.  
+
Consider what the ratio means. Since we know that they are consecutive terms, we can say
 +
<cmath>\frac{\dbinom{n}{k-1}}{3} = \frac{\dbinom{n}{k}}{4} = \frac{\dbinom{n}{k+1}}{5}.</cmath>
 +
 +
Taking the first part, and using our expression for <math>n</math> choose <math>k</math>,
 +
<cmath> \frac{n!}{3(k-1)!(n-k+1)!} = \frac{n!}{4k!(n-k)!}</cmath>
 +
<cmath> \frac{1}{3(k-1)!(n-k+1)!} = \frac{1}{4k!(n-k)!} </cmath>
 +
<cmath> \frac{1}{3(n-k+1)} = \frac{1}{4k} </cmath>
 +
<cmath> n-k+1 = \frac{4k}{3} </cmath>
 +
<cmath> n = \frac{7k}{3} - 1 </cmath>
 +
<cmath> \frac{3(n+1)}{7} = k </cmath>
 +
Then, we can use the second part of the equation.
 +
<cmath> \frac{n!}{4k!(n-k)!} = \frac{n!}{5(k+1)!(n-k-1)!} </cmath>
 +
<cmath> \frac{1}{4k!(n-k)!} = \frac{1}{5(k+1)!(n-k-1)!} </cmath>
 +
<cmath> \frac{1}{4(n-k)} = \frac{1}{5(k+1)} </cmath>
 +
<cmath> \frac{4(n-k)}{5} = k+1 </cmath>
 +
<cmath> \frac{4n}{5}-\frac{4k}{5} = k+1 </cmath>
 +
<cmath> \frac{4n}{5} = \frac{9k}{5} +1. </cmath>
 +
Since we know <math>k = \frac{3(n+1)}{7}</math> we can plug this in, giving us
 +
<cmath> \frac{4n}{5} = \frac{9\left(\frac{3(n+1)}{7}\right)}{5} +1 </cmath>
 +
<cmath> 4n = 9\left(\frac{3(n+1)}{7}\right)+5 </cmath>
 +
<cmath> 7(4n - 5) = 27n+27 </cmath>
 +
<cmath> 28n - 35 = 27n+27 </cmath>
 +
<cmath> n = 62 </cmath>
 +
We can also evaluate for <math>k</math>, and find that <math>k = \frac{3(62+1)}{7} = 27.</math> Since we want <math>n</math>, however, our final answer is <math>\boxed{062.}</math> ~LaTeX by ciceronii
  
After expanding and dividing one entry by another (to clean up the factorials), we see that <math>\displaystyle \frac 34=\frac{r+1}{n-r}</math> and <math>\displaystyle \frac45=\frac{r+2}{n-r-1}</math>. Solving, <math>\displaystyle n = \boxed{062}</math>
 
  
 +
==Solution 2==
 +
Call the row x, and the number from the leftmost side t. Call the first term in the ratio <math>N</math>. <math>N = \dbinom{x}{t}</math>. The next term is <math>N * \frac{x-t}{t+1}</math>, and the final term is <math>N * \frac{(x-t)*(x-t-1)}{(t+1)*(t+2)}</math>. Because we have the ratio, <math>N : N * \frac{x-t}{t+1} : N * \frac{(x-t)*(x-t-1)}{(t+1)*(t+2)}</math> = <math>3:4:5</math>,
  
== See also ==
+
<math>\frac{x-t}{t+1} = \frac{4}{3}</math> and <math>\frac{(x-t)*(x-t-1)}{(t+1)*(t+2)} = \frac{5}{3}</math>.
* [[1992 AIME Problems]]
+
Solve the equation to get get <math> t= 26 </math> and <math>x = \boxed{062}</math>.
 +
 
 +
https://www.wolframalpha.com/input/?i=%28x-t%29%2F%28t%2B1%29++%3D+4%2F3%2C++%28x-t%29%28x-t-1%29%2F%28%28t%2B1%29%28t%2B2%29%29+%3D+5%2F3
 +
 
 +
 
 +
-Solution and LaTeX by jackshi2006
 +
 
 +
 
 +
{{AIME box|year=1992|num-b=3|num-a=5}}
 +
 
 +
[[Category:Intermediate Combinatorics Problems]]
 +
[[Category:Intermediate Number Theory Problems]]
 +
{{MAA Notice}}

Revision as of 14:32, 22 November 2020

Problem

In Pascal's Triangle, each entry is the sum of the two entries above it. In which row of Pascal's Triangle do three consecutive entries occur that are in the ratio $3: 4: 5$?

Solution 1

Consider what the ratio means. Since we know that they are consecutive terms, we can say \[\frac{\dbinom{n}{k-1}}{3} = \frac{\dbinom{n}{k}}{4} = \frac{\dbinom{n}{k+1}}{5}.\]

Taking the first part, and using our expression for $n$ choose $k$, \[\frac{n!}{3(k-1)!(n-k+1)!} = \frac{n!}{4k!(n-k)!}\] \[\frac{1}{3(k-1)!(n-k+1)!} = \frac{1}{4k!(n-k)!}\] \[\frac{1}{3(n-k+1)} = \frac{1}{4k}\] \[n-k+1 = \frac{4k}{3}\] \[n = \frac{7k}{3} - 1\] \[\frac{3(n+1)}{7} = k\] Then, we can use the second part of the equation. \[\frac{n!}{4k!(n-k)!} = \frac{n!}{5(k+1)!(n-k-1)!}\] \[\frac{1}{4k!(n-k)!} = \frac{1}{5(k+1)!(n-k-1)!}\] \[\frac{1}{4(n-k)} = \frac{1}{5(k+1)}\] \[\frac{4(n-k)}{5} = k+1\] \[\frac{4n}{5}-\frac{4k}{5} = k+1\] \[\frac{4n}{5} = \frac{9k}{5} +1.\] Since we know $k = \frac{3(n+1)}{7}$ we can plug this in, giving us \[\frac{4n}{5} = \frac{9\left(\frac{3(n+1)}{7}\right)}{5} +1\] \[4n = 9\left(\frac{3(n+1)}{7}\right)+5\] \[7(4n - 5) = 27n+27\] \[28n - 35 = 27n+27\] \[n = 62\] We can also evaluate for $k$, and find that $k = \frac{3(62+1)}{7} = 27.$ Since we want $n$, however, our final answer is $\boxed{062.}$ ~LaTeX by ciceronii


Solution 2

Call the row x, and the number from the leftmost side t. Call the first term in the ratio $N$. $N = \dbinom{x}{t}$. The next term is $N * \frac{x-t}{t+1}$, and the final term is $N * \frac{(x-t)*(x-t-1)}{(t+1)*(t+2)}$. Because we have the ratio, $N : N * \frac{x-t}{t+1} : N * \frac{(x-t)*(x-t-1)}{(t+1)*(t+2)}$ = $3:4:5$,

$\frac{x-t}{t+1} = \frac{4}{3}$ and $\frac{(x-t)*(x-t-1)}{(t+1)*(t+2)} = \frac{5}{3}$. Solve the equation to get get $t= 26$ and $x = \boxed{062}$.

https://www.wolframalpha.com/input/?i=%28x-t%29%2F%28t%2B1%29++%3D+4%2F3%2C++%28x-t%29%28x-t-1%29%2F%28%28t%2B1%29%28t%2B2%29%29+%3D+5%2F3


-Solution and LaTeX by jackshi2006


1992 AIME (ProblemsAnswer KeyResources)
Preceded by
Problem 3
Followed by
Problem 5
1 2 3 4 5 6 7 8 9 10 11 12 13 14 15
All AIME Problems and Solutions

The problems on this page are copyrighted by the Mathematical Association of America's American Mathematics Competitions. AMC logo.png